Paedics Flashcards

1
Q

What are the major steps in taking paediatric history

A
Demographics 
Know who the primary care giver of the child is 
PC
HPC
ODQ
Systemic enquiry
Pregnancy and delivery history
Past medical and surgical history
Past drug history
Past family history
Immunization history
Developmental history 
Feeding and nutritional history 
Dietary recall
Social history 
Summary
How well did you know this?
1
Not at all
2
3
4
5
Perfectly
2
Q

What are you looking for in pregnancy and delivery history? And state why you’re looking for it
When asking,document everything you do and everything you asked the patient and if the patient doesn’t know,state that the patient didn’t know true or false
In history taking of a child,a child who is 12 years can be considered and an adult history will be taken for the child instead of paediatric history true or false

A

If the pregnancy was planned or not
(If pregnancy wasn’t planned,the child may suffer or person won’t get drugs to give the child )
If pregnancy was planned
What’s the duration of the pregnancy (in weeks not months)
Did the mother attend antenatal when prgansnt w the child?
Did she do all the tests asked?
Which tests did she do?(example urine ,blood etc like if patient had a G6PD deficiency)
What were the results of the test?
Did she take all the treatments given during the pregnancy (if person didn’t take tetanus vaccine the child won’t be protected against it)
During the pregnancy did the person get sick or Ill or was there any serious condition the woman had while pregnant?
After how many weeks was the child delivered
How was the child delivered
(If by Cs, ask why the CS was done)
What was the birth weight of the child
Did the child cry at birth or not? Was the cry delayed? (Resuscitation means did the child cry at birth)
Was the child jaundiced,cyanosed at birth?
Did the child pass mucous or stool or urine during the first 24 hours after birth?
Were there complications after delivery?
Was the child admitted after birth?
(State that delivery was uneventful if there were no complications)

How well did you know this?
1
Not at all
2
3
4
5
Perfectly
3
Q

In family history what do you ask?
In past med and surgical what do you ask?
In immunization history what do you ask?
In developmental history what do you ask
For developmental history
If the child is 10,11,12,14, you can decide not to ask about the months but if patient has had a problem from birth ,go into details

A

Fam:
Ask conditions that can be transferable to the child such as SCD, and other diseases

Past med hx and surgical:
If the child had any illness or has had any previous Illnesses or admissions
If Mum has illnesses
If there were blood transfusions

Immunization:
Did child get Al the vaccines
Ask which vaccines the child got
Request for the road to health chart or child health record book
( to report if immunization history is up to date for the age of the child)
Open the book to go through to confirm if the vaccination is up to date or not

Developmental:
What the child can do at his or her age

The milestones:
Gross motor development 
Fine motor development 
Social development 
Cognitive and mental development 

Gross motor:
At birth was the child moving the limbs or what could the child to that the person was able to see?
Flexed posture
Fine motor: Follows face or light by 2 weeks Follows moving object or face by turning the head (illustrated).
Hearing,speech and language or cognitive and mental ;
Stills to voice
Startles to loud noise
Social, emotional, and behavioural: smiles by 6 weeks

3-4 months: hearing speech and language or mental and cognitive: Vocalises alone or when spoken to, coos and laughs
4-6 months-
Gross motor:what could the child do at this point
Could child sit without support? With the back round?
Fine motor: Palmar grasp
Social: puts food in mouth
7 months- what could child do?
Could child sit without support?
Fine motor:
Could child pick up things?
Transfers objects from hand to hand
Mental and cognitive: Turns to soft sounds out of sight. At 7 months, sounds used indiscriminately.
Social- puts food in mouth

8 months- could child sit without support with back straight?
Could child crawl?
Social:puts food in mouth

9 months- what could child do?
Could child crawl?

10 months- Stands independently Cruises around furniture
Was child in crèche?
Fine motor: Mature pincer grip
Cognitive and mental: At 10 months, sounds used discriminately to parents
Social: Waves bye-bye, plays peek-a-boo

I year- could child walk,could child interact w others?
Walks unsteadily, broad gait, hands apart
Cognitive: Two to three words other than ‘dada’ or ‘mama’
Social: Drinks from a cup with two hands

15 months: Walks steadily
Fine motor: Immature grip of pencil
Random scribble

16-18 months - fine motor: Makes marks with a crayon. Immature grip of pencil,
Random scribble
Congnitive: 6–10 words. Shows two parts of the body
Social: Holds spoon and gets food safely to mouth. Beginning to help with dressing

20-24 months:cognitive: Joins two or more words to make simple phrases
Social: Clean and dry

2.5 to 3 years: cognitive : Talks constantly in 3–4 word sentences

Understands two joined commands

How well did you know this?
1
Not at all
2
3
4
5
Perfectly
4
Q

Who is a neonate, an infant,a toddler
A preschooler

2.2 The history must be adapted to the child’s
age. The age when a child first walks is highly relevant when taking the history of a toddler or child with a developmental problem but irrelevant for a teenager in secondary school with headaches.
True or false

Make sure that you and the parent or child mean the same thing when describing a problem. For example, parents may use the word ‘wheeze’ to describe any respiratory sound.
True or false

A
Neonate (<4 weeks) 
Infant (<1 year)
Toddler
Approx 1-2 years
Preschool
 (2-5 years)
How well did you know this?
1
Not at all
2
3
4
5
Perfectly
5
Q

What are you looking for in feeding and nutritional history

A

If mother breastfed the child
How long did she do the breastfeeding
Was it exclusive breastfeeding or it was mixed
If mixed,why?
What month was the child meaned?
How does she breastfeed if she breastfeeds?
Does she put breast milk in a bottle?
How does she clean the bottle?
Does she wash her hands before breastfeeding?
(If she’s still breastfeeding)
If woman uses formula,how does she clean the bottle ?
How many times is the child breastfed a day? (Child should not be fed less than 6-8 times a day)
If child doesn’t take breast milk or takes breast milk and normal food, ask what food the child eats?
Does the child eat adult food?
How many times does the child eat a day(child should eat more than thrice a day w snacks in between )
Components:
Proteins
How many times does the child eat meat or egg or fish in a day or a week
Minerals:
Does child like fruits? What kind of fruits? Do you give child fruits? How many times does child eat fruit in a day or in a week?
Carbs:
What food is given
Water:
How many times does child take after in a day
Dietary recall:
What has the child eaten previously? (24 hours minimum)
What food child eats at home not the hospital if child is admitted
If child has been admitted ask the day before admission what the child ate the whole day?
Ask what child ate and ask for each component w each dish said by woman
Between meals what did the child eat?
Morning afternoon and evening
If she gave Koko, ask if there was milk,sugar and koose or bread w egg ? Were there fruits?
Ask if child was getting balanced diet in each dish?
Find out what was used to make the stew if woman says rice and stew or something and stew?
If the child has malnutrition, do a week or two weeks dietary recall not 24 hours

How well did you know this?
1
Not at all
2
3
4
5
Perfectly
6
Q

What are you looking for in drug hx
What about social hx?
How is summary done

A
Drug:
If child is on medications 
Name of medications
What the doses are ?
Any allergies to food or medication?

Social hx:
Who is taking care of the child
Who does the child live w?
Where do they stay?
Is it their own home
What’s the ventilation status of where the child sleeps
How is refuse disposed
Source of drinking water and source of water
Average income of primary care giver
What work does he do?
What does mum use to cook cuz charcoal can aggravate asthma in kids
Occupation of parents; if mom is a farmer and stored kerosene ,the child can drink it and present w poisoning

Summary: Who you got the history fromz
Name
Age or age range(toddler,middle age)
Sex
All PCs and duration
Any negative or positive finding
How well did you know this?
1
Not at all
2
3
4
5
Perfectly
7
Q

What is the normal fetal heart rate

A

110-150bpm

How well did you know this?
1
Not at all
2
3
4
5
Perfectly
8
Q

How is examination of the newborn done

A
How well did you know this?
1
Not at all
2
3
4
5
Perfectly
9
Q

What are the nine most common congenital heart lesions

). Polygenic abnormalities probably explain why having a child with congenital heart disease doubles the risk for subsequent children and the risk is higher still if either parent has congenital heart disease. A small number are related to external teratogens.
True or false

A
Left-to-right shunts (breathless)
• Ventricular septal defect 30%
• Persistent arterial duct 12%
• Atrial septal defect 7%
Right-to-left shunts (blue)
• Tetralogy of Fallot 5%
• Transposition of the great arteries 5%
Common mixing (breathless and blue)
• Atrioventricular septal defect (complete) 2%
Outflow obstruction in a well child (asymptomatic with a murmur)
• Pulmonary stenosis 7%
• Aortic stenosis 5%
Outflow obstruction in a sick neonate (collapsed with shock)
• Coarctation of the aorta 5%.
How well did you know this?
1
Not at all
2
3
4
5
Perfectly
10
Q

What are the circulatory changes at birth

A

In the fetus, the left atrial pressure is low, as relatively little blood returns from the lungs. The pressure in the right atrium is higher than in the left, as it receives all the systemic venous return including blood from the placenta. The flap valve of the foramen ovale is held open, blood flows across the atrial septum into the left atrium, and then into the left ventricle, which in turn pumps it to the upper body (Figs 18.1 and 10.8).
With the first breaths, resistance to pulmonary blood flow falls and the volume of blood flowing through the lungs increases six-fold. This results in a rise in the left atrial pressure. Meanwhile, the volume of blood return- ing to the right atrium falls as the placenta is excluded from the circulation. The change in the pressure
Table 18.1 Causes of congenital heart disease
difference causes the flap valve of the foramen ovale to be closed. The ductus arteriosus, which connects the pulmonary artery to the aorta in fetal life, will normally close within the first few hours or days. Some babies with congenital heart lesions rely on blood flow through the duct (duct-dependent circulation). Their clinical condition will deteriorate dramatically when the duct closes, which is usually at 1–2 days of age but occasionally later.

How well did you know this?
1
Not at all
2
3
4
5
Perfectly
11
Q

State the causes of congenital heart diseases under maternal disorders ,maternal drugs and chromosomal abnormalities and state the congenital diseases they cause

A
Maternal disorders:
  Rubella infection-
Peripheral pulmonary stenosis, PDA
Systemic lupus erythematosus-
Complete heart block (anti-Ro and anti-La antibody)
Maternal drugs:
Warfarin therapy-
Pulmonary valve stenosis, PDA
Fetal alcohol syndrome-
ASD, VSD, tetralogy of Fallot
Chromosomal abnormality:
Down syndrome (trisomy 21)-
Atrioventricular septal defect, VSD

Edwards syndrome (trisomy 18)-
Complex congenital heart disease
Patau syndrome (trisomy 13)-
Complex congenital heart disease

Turner syndrome (45XO)-
Aortic valve stenosis, coarctation of the aorta

Chromosome 22q11.2 deletion-
Aortic arch anomalies, tetralogy of Fallot, common arterial trunk

Williams syndrome (7q11.23 microdeletion)-
Supravalvular aortic stenosis, peripheral pulmonary artery stenosis
Noonan syndrome (PTPN11 mutation and others)-
Hypertrophic cardiomyopathy, atrial septal defect, pulmonary valve stenosis
How well did you know this?
1
Not at all
2
3
4
5
Perfectly
12
Q

Congenital heart diseases presents with which signs

At what gestational age is fetal anomaly scan done

A
Congenital heart disease presents with:
• antenatalcardiacultrasounddiagnosis • detectionofaheartmurmur
• heartfailure
• shock
• cyanosis.

18 weeks’ and 20 weeks’ gestation

How well did you know this?
1
Not at all
2
3
4
5
Perfectly
13
Q

Features of GiT disorders in kids are of what signs?
What is posseting and regurgitation?
Ehat do they indicate?
What is vomiting ? It is usually benign and is often caused by feeding disorders or mild gastro-oesophageal reflux or gastroenteritis. True or false

In infants, vomiting may be associated with infection outside the gastrointestinal tract, especially
Which systems?

intestinal obstruction, the more proximal the obstruc- tion, the more prominent the vomiting and the sooner it becomes bile stained (unless the obstruction is proxi- mal to the ampulla of Vater). Intestinal obstruction is associated with abdominal distension, more marked in distal obstruction. ‘
True or false
What is the characteristic of vomit in intestinal obstruction,(Oesophagitis, peptic ulceration, oral/nasal bleeding, and oesophageal variceal bleeding),pyloric stenosis,whooping cough or pertussis, what signs are associated with vomiting in surgical abdomen,Intestinal obstruction(including strangulated inguinal hernia), Chronic liver disease, inborn error of metabolism Intussusception, bacterial gastroenteritis
Severe gastroenteritis, systemic infection (urinary tract infection, meningitis), diabetic ketoacidosis) ,
Raised intracranial pressure,
Gastro-oesophageal reflux disease, coeliac disease and other chronic gastrointestinal conditions

Name six causes of vomiting in infants, preschool children, school age and adolescents

What’s a common chronic cause of vomiting in infants?
Over feeding is common in which people?
When will you suspect gastroenteritis in infants but what else should you consider?
If vomit is projectile at 2-8 weeks of age what should you exclude?
If vomit is bile stained it’s a potential emergency so exclude what diseases and assess for what?

A

• vomiting,abdominalpainanddiarrhoeaa that are transient
(lasting only for a short time)

Posseting and regurgitation are terms used to describe the non-forceful return of milk, but differ in degree. Posseting describes the small amounts of milk that often accompany the return of swallowed air (wind), whereas regurgitation describes larger, more frequent losses. Posseting occurs in nearly all babies from time to time, whereas regurgitation may indicate the pres- ence of more significant gastro-oesophageal reflux.
Vomiting is the forceful ejection of gastric contents. It is a common problem in infancy and childhood

True

urinary tract and central nervous system.

True

Intestinal obstruction:bile stained vomit
Oesophagitis, peptic ulceration, oral/nasal bleeding, and oesophageal variceal bleeding): hematemesis
Pyloric stenosis: Projectile vomiting, in first few weeks of life
Vomiting at the end of paroxysmal coughing: whooping cough
Abdominal tenderness/abdominal pain on movement:surgical abdomen
Abdominal distension: intestinal obstruction
Hepatosplenomegaly : Chronic liver disease, inborn error of metabolism
Blood in the stool: Intussusception, bacterial gastroenteritis
Severe dehydration, shock: Severe gastroenteritis, systemic infection (urinary tract infection, meningitis), diabetic ketoacidosis,
Bulging fontanelle or seizures : Raised intracranial pressure,
Faltering growth: Gastro-oesophageal reflux disease, coeliac disease and other chronic gastrointestinal conditions

Infants
Gastro-oesophageal reflux Feeding problems Infection:
• Gastroenteritis
• Respiratory tract/otitis media
• Whooping cough (pertussis)
• Urinary tract
• Meningitis
Food allergy and food intolerance Eosinophilic oesophagitis Intestinal obstruction:
• Plyoric stenosis
• Atresia – duodenal,
other sites
• Intussusception
• Malrotation
• Volvulus
• Duplication cysts
• Strangulated inguinal
hernia
• Hirschsprung disease Inborn errors of metabolism 
Congenital adrenal hyperplasia 
Renal failure
Preschool children
Gastroenteritis
Infection:
• Respiratory tract/otitis
media
• Urinary tract
• Meningitis
• Whooping cough (pertussis) Appendicitis
Intestinal obstruction:
• Intussusception
• Malrotation
• Volvulus
• Adhesions
• Foreign body – bezoar 
Raised intracranial pressure 
Coeliac disease
Renal failure
Inborn errors of metabolism 
Torsion of the testis
School age and adolescents
Gastroenteritis
Infection – including pyelonephritis, septicaemia, meningitis
Peptic ulceration and H. pylori infection Appendicitis
Migraine
Raised intracranial pressure
Coeliac disease
Renal failure
Diabetic ketoacidosis
Alcohol/drug ingestion or medications Cyclical vomiting syndrome Bulimia/anorexia nervosa
Pregnancy
Torsion of the testis

Vomiting in infants
• Common chronic cause is gastro-oesophageal reflux.
• Feed volumes should be calculated as overfeeding is common in bottle-fed infants.
• If transient, with other symptoms, e.g. fever, diarrhoea or runny nose and cough, most likely to be gastroenteritis or respiratory tract infection, but consider urine infection, sepsis or meningitis.
• If projectile at 2–8 weeks of age, exclude pyloric stenosis.
• If bile stained, potential emergency – exclude intestinal obstruction, especially intussusception, malrotation and a strangulated inguinal hernia. Assess for dehydration and shock.

How well did you know this?
1
Not at all
2
3
4
5
Perfectly
14
Q
What is GER? 
Which people is it common in? What causes it? Name three things that contribute to GER? While common in the 1st year of life, nearly all symptomatic reflux resolves spontaneously by 12 months of age. True or false and why? Most infants w GER have what? What happens when GER becomes a significant problem?
GERD is common in which kids?
Name some complications of GERD in kids
How do you manage GERD
A

Gastro-oesophageal reflux
Gastro-oesophageal reflux is the involuntary passage of gastric contents into the oesophagus. It is extremely common in infancy. It is caused by inappropriate relaxation of the lower oesophageal sphincter as a result of functional immaturity. A predominantly fluid diet, a mainly horizontal posture and a short intra- abdominal length of oesophagus all contribute. While common in the 1st year of life, nearly all symptomatic reflux resolves spontaneously by 12 months of age. This is probably due to a combination of maturation of the lower oesophageal sphincter, assumption of an upright posture and more solids in the diet.
Most infants with gastro-oesophageal reflux have recurrent regurgitation or vomiting but are putting on weight normally and are otherwise well, although the mess, smell, and frequent changes of clothes (5% of those affected have 6 or more episodes each day) is frustrating for parents and carers.
Gastro-oesophageal reflux is usually a benign, self- limited condition but when it becomes a significant problem it becomes gastro-oesophageal reflux disease and needs treatment.

Gastro-oesophageal reflux disease is more common in:
• childrenwithcerebralpalsyorother neurodevelopmental disorders
• preterm infants, especially in those with bronchopulmonary dysplasia
• following surgery for oesophageal atresia or diaphragmatic hernia.
-neuromuscular problems or has had surgery to the oesophagus or diaphragm.

Complications of gastro-oesophageal reflux (i.e. gastro-oesophageal reflux disease)
• Faltering growth from severe vomiting
• Oesophagitis – haematemesis, discomfort on
feeding or heartburn, iron-deficiency anaemia
• Recurrent pulmonary aspiration – recurrent
pneumonia, cough or wheeze, apnoea in
preterm infants
• Dystonic neck posturing (Sandifer syndrome)
• Apparent life-threatening events

Management:
Uncomplicated gastro-oesophageal reflux has an excellent prognosis and can be managed by parental reassurance, adding inert thickening agents to feeds (e.g. Carobel), and smaller, more frequent feeds.
Significant gastro-oesophageal reflux disease is managed with acid suppression with either hydrogen receptor antagonists (e.g. ranitidine) or proton-pump inhibitors (e.g. omeprazole). These drugs reduce the volume of gastric contents and treat acid-related oesophagitis. The evidence for the use of drugs that enhance gastric emptying (e.g. domperidone) is poor and as they are associated with significant side-effects their use should be discouraged. If the child fails to respond to these measures, other diagnoses such as cow’s milk protein allergy should be considered and further investigations performed.
Surgical management is reserved for children with complications unresponsive to intensive medical treat- ment or oesophageal stricture. A Nissen fundoplica- tion, in which the fundus of the stomach is wrapped around the intra-abdominal oesophagus, is performed either as an abdominal or as a laparoscopic procedure.

How well did you know this?
1
Not at all
2
3
4
5
Perfectly
15
Q

In pyloric stenosis what causes gastric outlet obstruction? When does it present irrespective of gestational age?
It is common in which sec? Particularly which people and there may be a fam history on which side of the family?
What’s re the clinical features?
What occurs as a result of vomiting stomach contents?
In diagnosis of pyloric stenosis unless what is required what is performed?
Gastric peristalsis may be seen as what? The pyloric mass is usually palpable where? If the stomach is overdistended with air what must be done? What exam may be helpful to confirm diagnosis prior to surgery?
What’s the management of pyloric stenosis
What are the signs of pyloric stenosis?

A

In pyloric stenosis, there is hypertrophy of the pyloric muscle causing gastric outlet obstruction. It presents at 2–8 weeks of age, irrespective of gestational age. It is more common in boys (4 : 1), particularly firstborn, and there may be a family history, especially on the maternal side.
Clinical features are:
• vomiting, which increases in frequency and forcefulness over time, ultimately becoming projectile
• hunger after vomiting until dehydration leads to loss of interest in feeding
• weight loss if presentation is delayed.

A hypochloraemic metabolic alkalosis with a low plasma sodium and potassium occurs as a result of vomiting stomach contents.

Diagnosis
Unless immediate fluid resuscitation is required, a test feed is performed. The baby is given a milk feed, which will calm the hungry infant, allowing examination. Gastric peristalsis may be seen as a wave moving from left to right across the abdomen (Fig. 14.3a). The pyloric mass, which feels like an olive, is usually palpable in the right upper quadrant (Fig. 14.3b). If the stomach is overdistended with air, it will need to be emptied by a nasogastric tube to allow palpation. Ultrasound examination may be helpful (Fig. 14.3c) to confirm the diagnosis prior to surgery.
Management
The initial priority is to correct any fluid and electrolyte disturbance with intravenous fluids. Once hydration and acid–base and electrolytes are normal, definitive treatment by pyloromyotomy can be performed. This involves division of the hypertrophied muscle down to, but not including, the mucosa (Fig. 14.3d). The opera- tion can be performed either as an open procedure via a periumbilical incision or laparoscopically. Postopera- tively, the child can usually be fed within 6 hours and discharged within 2 days of surgery.
• Signs are visible gastric peristalsis, palpable abdominal mass on test feed, and possible dehydration.
• Associated with hyponatraemia, hypokalaemia, and hypochloraemic alkalosis.
• Diagnosis may be confirmed by ultrasound.
• Treated by surgery after rehydration and
correction of electrolyte imbalance.

How well did you know this?
1
Not at all
2
3
4
5
Perfectly
16
Q

Crying in healthy babies represents baby’s response to what?
What complain may also be a pointer to potential or actual non-accidental injury.? If there a sudden onset of crying what may it be due to?
Why can cause miserable crying infant?
What is infant colic? Paroxysmal inconsolable crying or screaming often accompanied by what can take place is infant colic? When does it typically occur? If the colic is severe an persistent it may be due to what?

A

In most, it represents the baby’s response to hunger and discomfort. Reassurance and advice on appropriate feeding will usually suffice.
Some babies cry for prolonged periods in spite of feeding and comforting and this is distressing for all concerned. It can engender a feeling of anxiety, helplessness and depression in parents and carers, par- ticularly if they are inexperienced or poorly supported. It has also been suggested that the emotional climate within a home may be transmitted to a baby, and that in some instances, tense, anxious, or irritable caregivers are more likely to have fretful babies. The complaint that a baby is ‘always crying’ may also be a pointer to potential or actual non-accidental injury.
A cause for the crying is identified in a minority of infants. If of sudden onset, it may be due to a urinary tract, middle ear or meningeal infection; pain from an unrecognized fracture; oesophagitis; or torsion of the testis. Severe nappy rash and constipation may produce a miserable, crying infant. Preterm infants who have spent several weeks in hospital can be difficult to settle, as can infants with a chronic neurological disorder, e.g. cerebral palsy. On the basis of countless reports of parents, eruption of teeth is painful in some infants. However, teething does not cause vomiting, diarrhoea, high fever or seizures.
Acknowledging that troublesome crying is extremely distressing is part of the management. Reducing overstimulation from jigging and winding and encouraging a quiet environment and holding the baby close until the crying stops appear to help many babies.

Infant ‘colic’
The term ‘colic’ is used to describe a common symptom complex that occurs during the first few months of life. Paroxysmal, inconsolable crying or screaming often accompanied by drawing up of the knees and passage of excessive flatus takes place several times a day. There is no firm evidence that the cause is gastrointestinal, but this is often suspected. The condition occurs in up to 40% of babies. It typically occurs in the first few weeks of life and resolves gradually from 3–12 months of age. The condition is benign but it is very frustrat- ing and worrying for parents and may precipitate non-accidental injury in infants already at risk. Support and reassurance should be given. ‘Gripe water’ is often recommended but is of unproven benefit. If severe and persistent, it may be due to a cow’s milk protein allergy and an empirical 2-week trial of a protein hydrolysate formula (cow’s milk protein free) may be considered and continued if symptoms improve. If they do not, then a trial of gastro-oesophageal reflux treatment may be considered.

How well did you know this?
1
Not at all
2
3
4
5
Perfectly
17
Q

What are the causes of acute abdominal pain?
Which cause of acute abdominal pain surgically is most common
Which organs must always be checked in acute abdominal pain ?
Although it may occur at any age, it is very uncommon in children of how many years? What are the clinical features of acute uncomplicated appendicitis?(state four signs and four symptoms don’t forget to characterize pain)
What may cause referred pain to the abdomen?
Primary peritonitis is seen in which patients?
What sugar related disease can cause severe abdominal pain? What are two uncommon causes of acute abdominal pain that must not be missed. Why will it be important to test the urine sample ? Pancreatitis may present w acute abdominal pain and what marker should be checked?

Appendicitis is a progressive condition and so repeated observation and clinical review every few hours are key to making the correct diagnosis, avoiding delay on the one hand and unnecessary laparotomy on the other. True or false
Give some signs of complicated appendicitis
In Acute abdominal pain in older children and adolescents wha causes should you exclude? What should you check for in boys ? What is seen when you palpate the abdomen of kids w acute appendicitis? How do you distinguish between acute appendicitis and non-specific abdominal pain ?
When is laparotomy indicated?

A
Surgical:
   Acute appendicitis
Intestinal obstruction including
intussusception
Inguinal hernia
Peritonitis
Inflamed Meckel diverticulum Pancreatitis
Trauma
From surgical to Intra-abdominal to medical :
 Non-specific abdominal pain Gastroenteritis
Urinary tract:
• urinary tract infection
• acute pyelonephritis
• hydronephrosis
• renal calculus Henoch–Schönlein purpura Diabetic ketoacidosis Sickle cell disease
Hepatitis
Inflammatory bowel disease Constipation
Recurrent abdominal pain of
childhood
Gynaecological in pubertal females Psychological
Lead poisoning
Acute porphyria (rare)
Unknown
 Extra-abdominal:

Upper respiratory tract infection Lower lobe pneumonia
Torsion of the testis
Hip and spine

Appendicitis

The testes, hernial orifices and hip joints must always be checked.

  • lower lobe pneumonia may cause pain referredto the abdomen
  • primary peritonitis Is seen in patients with ascites from nephrotic syndrome or liver disease
  • diabeticketoacidosismaycausesevereabdominal pain
  • urinary tract infection, including acute pyelonephritis, is a relatively uncommon cause of acute abdominal pain, but must not be missed. It is important to test a urine sample, in order to identify not only diabetes mellitus but also conditions affecting the liver and urinary tract
  • pancreatitismaypresentwithacuteabdominal pain and serum amylase should be checked.

Acute appendicitis
Acute appendicitis is the most common cause of abdominal pain in childhood requiring surgical inter- vention .Although it may occur at any age, it is very uncommon in children under 3 years of age. The clinical features of acute uncomplicated appen- dicitis are:
• Symptoms
– Anorexia
– Vomiting
– Abdominal pain, initially central and colicky (appendicular midgut colic), but then localizing to the right iliac fossa (from localized peritoneal
• inflammation)
Signs
– Fever
– Abdominal pain aggravated by movement, e.g.
on walking, coughing, jumping, bumps on the
road during a car journey
– Persistent tenderness with guarding in the right
iliac fossa (McBurney’s point). However, with a retrocaecal appendix,localized guarding may be absent, and in a pelvic appendix there may be few abdominal signs.

Appendicectomy is straightforward in uncompli- cated appendicitis. Complicated appendicitis includes the presence of an appendix mass, an abscess, or perforation. If there is generalized guarding consistent with perforation, fluid resuscitation and intravenous antibiotics are given prior to laparotomy. If there is a palpable mass in the right iliac fossa and there are no signs of generalized peritonitis, it may be reasonable to elect for conservative management with intravenous antibiotics, with appendicectomy being performed after several weeks. If symptoms progress, laparotomy is indicated.

Acute abdominal pain in older children and adolescents
• Exclude medical causes, in particular lower lobe
pneumonia, diabetic ketoacidosis, hepatitis, and
pyelonephritis.
• Check for strangulated inguinal hernia or
torsion of the testis in boys.
• On palpating the abdomen in children with
acute appendicitis, guarding and rebound tenderness are often absent or unimpressive, but pain from peritoneal inflammation may be demonstrated on coughing, walking or jumping.
• To distinguish between acute appendicitis and non-specific abdominal pain may require close monitoring, joint management between paediatricians and paediatric surgeons and repeated evaluation in hospital.

How well did you know this?
1
Not at all
2
3
4
5
Perfectly
18
Q

What is non specific abdominal pain?
What is it often accompanied by? Mesenteric adenitis is often diagnosed in which children?
What’s intussusception?
It most commonly involves which part of the GIT? Intussusception is the most common cause of what in infants after the neonatal period? What’s the peak age of presentation? What’s the most serious complication? What does it result in? What is essential to avid complication?
Presentation in intussusception is typically with what?
How is X ray and abdominal ultrasound of the abdomen helpful?
When is surgery required?
Reduction of intussusception is attempted how ? What’s an important complication?
What are the clinical features? Usually occurs in children of what age?

A

Non-specific abdominal pain and mesenteric adenitis
Non-specific abdominal pain is abdominal pain which resolves in 24–48 hours. The pain is less severe than in appendicitis, and tenderness in the right iliac fossa is variable. It is often accompanied by an upper respira- tory tract infection with cervical lymphadenopathy. In some of these children, the abdominal signs do not resolve and an appendicectomy is performed. Mesenteric adenitis is often diagnosed in those children in whom large mesenteric nodes are seen at laparoscopy and whose appendix is normal, but there are doubts whether this condition truly exists as a diagnostic entity.

Intussusception
Intussusception describes the invagination of proxi- mal bowel into a distal segment. It most commonly involves ileum passing into the caecum through the ileocaecal valve (Fig. 14.6a). Intussusception is the most
common cause of intestinal obstruction in infants after the neonatal period. Although it may occur at any age, the peak age of presentation is 3 months – 2 years of age. The most serious complication is stretching and constriction of the mesentery resulting in venous obstruction, causing engorgement and bleeding from the bowel mucosa, fluid loss, and subsequently bowel perforation, peritonitis and gut necrosis. Prompt diagnosis, immediate fluid resuscitation and urgent reduction of the intussusception are essential to avoid complications.
Presentation is typically with:
• Paroxysmal, severe colicky pain with pallor
– during episodes of pain, the child becomes pale, especially around the mouth, and draws up the legs. There is recovery between the painful episodes but subsequently the child may become increasingly lethargic.
• Mayrefusefeeds,mayvomit,whichmaybecome bile stained depending on the site of the intussusception.
• Asausage-shapedmass–oftenpalpableinthe abdomen
• Passageofacharacteristicredcurrantjellystool comprising blood-stained mucus – this is a characteristic sign but tends to occur later in the illness and may be first seen after a rectal examination.
• Abdominaldistensionandshock.
Usually, no underlying intestinal cause for the intus- susception is found, although there is some evidence that viral infection leading to enlargement of Peyer’s patches may form the lead point of the intussusception. An identifiable lead point such as a Meckel diverticulum or polyp is more likely to be present in children over 2 years of age. Intravenous fluid resuscitation is likely to be required immediately, as there is often pooling of fluid in the gut, which may lead to hypovolaemic shock.

An X-ray of the abdomen may show distended small bowel and absence of gas in the distal colon or rectum. Sometimes the outline of the intussusception itself can be visualized. Abdominal ultrasound is helpful both to confirm the diagnosis (the so-called target/doughnut sign) and to check response to treatment.

Intussusception
• Usually occurs between 3 months and 2 years of age.
• Clinical features are paroxysmal, colicky pain with pallor, abdominal mass and redcurrant jelly stool.
• Shock is an important complication and requires urgent treatment.
• Reduction is attempted by rectal air insufflation unless peritonitis is present.
• Surgery is required if reduction with air is unsuccessful or for peritonitis.

19
Q

What is malrotation?
Ladd bands?
What are the two presentations?
What is the usual presentation in the first few days of life? Any child with dark green vomiting needs and urgent upper gastrointestinal contrast study to assess intestinal rotation, unless signs of vascular compromise are present, when an urgent laparotomy is needed. Why is this a surgical emergency? At operation what is done?
Malrotation May present at any age with what? What are the clinical features?

A

Malrotation
During rotation of the small bowel in fetal life, if the mesentery is not fixed at the duodenojejunal flexure or in the ileocaecal region, its base is shorter than normal, and is predisposed to volvulus. Ladd bands are peritoneal bands that may cross the duodenum, often anteriorly .
There are two presentations:
• obstruction
•obstructionwithacompromisedbloodsupply.
Obstruction with bilious vomiting is the usual presen- tation in the first few days of life but can be seen at a later age. This is a surgical emergency as, when a volvulus occurs, the superior mesenteric arterial blood supply to the small intestine and proximal large intestine is compromised and unless it is corrected will lead to infarction of these areas.
At operation, the volvulus is untwisted, the duo- denum mobilized, and the bowel placed in the non- rotated position with the duodenojejunal flexure on the right and the caecum and appendix on the left. The malrotation is not ‘corrected’, but the mesentery broadened. The appendix is generally removed to avoid diagnostic confusion should the child subsequently have symptoms suggestive of appendicitis.

Summary
Malrotation
• Uncommon but important to diagnose.
• Usually presents in the first 1–3 days of life with
intestinal obstruction from Ladd bands
obstructing the duodenum or volvulus.
• May present at any age with volvulus causing
obstruction and ischaemic bowel.
• Clinical features are bilious vomiting, abdominal
pain and tenderness from peritonitis or
ischaemic bowel.
• An urgent upper gastrointestinal contrast study
is indicated if there is bilious vomiting. • Treatment is urgent surgical correction.

20
Q

What is recurrent abdominal pain
What is the characteristic of this pain?
What is a frequent cause of this pain?
Why can anxiety be a cause if altered bowel motility leading to recurrent abdominal pain.

It is increasingly recognized that many will have distinct symptom constellations resulting from func- tional abnormalities of gut motility – irritable bowel syndrome (most common), constipation, and less commonly coeliac disease, abdominal migraine and functional dyspepsia. True or false
How is recurrent abdominal pain managed
What is the prognosis for someone w this pain
What are the possible organic causes of recurrent abdominal pain(group them under GI,gynecology,hepatobiliary,urinary,psychosocial
State the signs of organic cause of recurrent abdominal pain

A

Recurrent abdominal pain is a common childhood problem. It is often defined as pain sufficient to inter- rupt normal activities and lasts for at least 3 months. The pain is characteristically periumbilical and the children are otherwise entirely well. Constipation is a frequent cause of recurrent abdominal pain and must be excluded. There is evidence that anxiety may lead to altered bowel motility, which may be perceived by the child as pain.

To do this, a full history and thorough examina- tion is required, which includes inspection of the perineum for anal fissures. The child’s growth should be checked.
A urine microscopy and culture is mandatory as urinary tract infections may cause pain in the absence of other symptoms or signs. An abdominal ultrasound is particularly helpful in excluding gall stones and pelvi- ureteric junction obstruction.
Although there are many potential organic causes, most are rare. Coeliac antibodies and thyroid function tests should be checked, but further investigations should be performed only if clinically indicated.
With irritable bowel syndrome and functional dys- pepsia, it can be helpful to explain to both the child and parents that ‘sometimes the insides of the intestine become so sensitive that some children can feel the food going round the bends’.
The long-term prognosis is that:
• about half of affected children rapidly become free of symptoms
• in one-quarter, the symptoms take some months to resolve
• in one-quarter, symptoms continue or return in adulthood as migraine, irritable bowel syndrome or functional dyspepsia.

Gastrointestinal
• Irritable bowel syndrome
• Constipation
• Non-ulcer dyspepsia
• Abdominal migraine
• Gastritis and peptic ulceration
• Eosinophilic oesophagitis
• Inflammatory bowel disease
• Malrotation

Gynaecological
• Dysmenorrhoea
• Ovarian cysts
• Pelvic inflammatory disease

Psychosocial – bullying, abuse, stress, etc. – a small proportion

Hepatobility/pancreatic
• Hepatitis
• Gall stones
• Pancreatitis

Urinary tract
• Urinary tract infection
• Pelvi-ureteric junction (PUJ) obstruction

Symptoms and signs that suggest organic disease:
• Epigastric pain at night, haematemesis (duodenal ulcer)
• Diarrhoea, weight loss, growth failure, blood in stools (inflammatory
bowel disease)
• Vomiting (pancreatitis)
• Jaundice (liver disease)
• Dysuria, secondary enuresis (urinary tract infection)
• Bilious vomiting and abdominal distension (malrotation)

21
Q

Abdominal migraine is often associated with what? Where’s the site of the attacks and what’s it usually associated with? What is the characteristic history you’ll see?
Irritable bowel syndrome is associated with what? Symptoms may be precipitated by what? What are the characteristic set of symptoms in IBS
Some kids and adults w IBS symptoms have what disease ?

A

Abdominal migraine
Abdominal migraine is often associated with abdomi- nal pain in addition to headaches, and in some children the abdominal pain predominates. The attacks of abdominal pain are midline associated with vomiting and facial pallor. There is usually a personal or family history of migraine. The history is characteristic with long periods (often weeks) of no symptoms and then a shorter period (12–48 hours) of non-specific abdominal pain and pallor, with or without vomiting

Irritable bowel syndrome
This disorder, also common in adults, is associated with altered gastrointestinal motility and an abnormal sen- sation of intra-abdominal events. Symptoms may be precipitated by a gastro-intestinal infection.
There is often a positive family history and a char- acteristic set of symptoms, although not all patients experience every symptom:
• non-specific abdominal pain, often peri-umbilical, may be worse before or relieved by defaecation
• explosive, loose, or mucousy stools
• bloating
• feeling of incomplete defecation
• constipation (often alternating with normal or
loose stools).
Some children and adults with irritable bowel symptoms have coeliac disease, which is why coeliac antibody serology must be checked.

22
Q

H pylori gram negative organism in association w what kind of gastritis is a potential cause of abdominal pain in kids? HPylori is a strong predisposing factor to what ulcers? These ulcers are uncommon but should be considered in which people ?
H pylori causes what kind of gastritis associated with what? When is it usually identified ? What does the organism produce and what does it form the basis for? Which tests are less reliable in young kids? Kids in whom peptic ulceration is suspected should be treated w what? Those who don’t respond to treatment or whose symptoms recur on stopping treatment should have what investigation and the outcome of the investigation will rule in our rule out what disease? What are the symptoms of functional dyspepsia?
What is eosinophilia oesophagitis? How does it present? It is more common in which kids? Diagnosis is by what? Treatment is with what?

A

Peptic ulceration, gastritis, and functional dyspepsia
The the iden- tification of the Gram-negative organism Helicobacter pylori in association in antral gastritis . H. pylori is a strong predis- posing factor to duodenal ulcers. Duodenal ulcers are uncommon in children but should be considered in those with epigastric pain, particularly if it wakes them at night, if the pain radiates through to the back, or when there is a history of peptic ulceration in a first- degree relative.
H. pylori causes a nodular antral gastritis, which may be associated with abdominal pain and nausea. It is usually identified in gastric antral biopsies. The organism produces urease, which forms the basis for a laboratory test on biopsies and the 13C breath test following the administration of 13C-labelled urea by mouth. Stool antigen for H. pylori may be positive in infected children. Serological tests are less reliable in young children but may be helpful in older children.
Children in whom peptic ulceration is suspected should be treated with proton-pump inhibitors, e.g. omeprazole, and if investigations suggest they have an H. pylori infection, eradication therapy should be given (amoxicillin and metronidazole or clarithromycin). Those who fail to respond to treatment or whose symptoms recur on stopping treatment should have an upper gastrointestinal endoscopy and, if this is normal, functional dyspepsia is diagnosed. Functional dyspepsia is probably a variant of irritable bowel syndrome.
As well as having symptoms of peptic ulceration, children with functional dyspepsia have rather more non-specific symptoms, including early satiety, bloat- ing, and postprandial vomiting and may have delayed gastric emptying as a result of gastric dysmotility. Treatment is difficult but some children respond to a hypoallergenic diet.

Eosinophilic oesophagitis
Eosinophilic oesophagitis is an inflammatory condi- tion affecting the oesophagus caused by activation of eosinophils within the mucosa and submucosa.
It can present with vomiting, discomfort on swal- lowing or bolus dysphagia, when food “sticks in the upper chest”. It is probably an allergic phenomenon although the precise pathophysiology is unclear. It is more common in children with other features of atopy (asthma, eczema, and hay fever). Diagnosis is by endoscopy where macroscopically, linear furrows and trachealization of the oesophagus may be seen, and microscopically, eosinophilic infiltration is identified. Treatment is with swallowed corticosteroids in the form of fluticasone or viscous budesonide. Exclusion diets may be of benefit in young children.

23
Q

What is a major cause of child mortality in developing countries
What’s the most frequent cause of gastroenteritis in developed countries?
Name two viruses that can cause outbreaks. If bacteria is a cause of the gastroenteritis it will be seen by the presence of what sign? What bacterial infection is often associated w severe abdominal pain? Which ones produce a dysenteries type of infection w blood and pus in the stool,pain and tenesmus. Which may be accompanied w high fever? Which are associated w profuse rapidly dehydrating diarrhea?
A protozoan cause of gastroenteritis is what organisms? What sign will you see in gastroenteritis? In the social history what may show its gastroenteritis?
What is the most serious complication of gastroenteritis?
Which kids are at increased risk of dehydration? Why are infants at a particular risk of dehydration?

A

gastroenteritis remains a major cause of child mortality.
The most frequent cause of gastroenteritis in devel- oped countries is rotavirus infection, which accounts for cases in children under 2 years of age.Other viruses, particularly adenovirus, norovirus, calici- virus, coronavirus and astrovirus may cause outbreaks
Bacterial causes are less common in developed countries but may be suggested by the presence of blood in the stools. Campylobacter jejuni infection, the most common of the bacterial infections in developed countries, is often associated with severe abdominal pain. Shigella and some salmonellae produce a dysenteric type of infection, with blood and pus in the stool, pain and tenesmus. Shigella infection may be accompanied by high fever. Cholera and entero- toxigenic Escherichia coli infection are associated with profuse, rapidly dehydrating diarrhoea. However, clinical features act as a poor guide to the pathogen.
The third cause of gastroenteritis is protozoan parasite infection such as Giardia and Cryptosporidium. In gastroenteritis there is a sudden change to loose or watery stools often accompanied by vomiting. There may be contact with a person with diarrhoea and/or vomiting or recent travel abroad. Dehydration leading to shock is the most serious com- plication and its prevention or correction is the main
aim of treatment.
The following children are at increased risk of
dehydration:
• infants, particularly those under 6 months of age or those born with low birthweight
• if they have passed six or more diarrhoeal stools in the previous 24 hours
• if they have vomited three or more times in the previous 24 hours
• if they have been unable to tolerate (or not been offered) extra fluids
• if they have malnutrition.
Infants are at particular risk of dehydration because they have a greater surface area-to-weight ratio than older children, leading to greater insensible water losses (300 ml/m2 per day, equivalent in infants to 15–17 ml/kg per day). They also have higher basal fluid requirements (100–120 ml/kg per day, i.e. 10% to 12% of bodyweight) and immature renal tubular reabsorp- tion. In addition, they are unable to obtain fluids for themselves when thirsty.

24
Q

What is the most accurate measure of dehydration? What can cause the measurement to be misleading? How are history and examination used to assess the degree of dehydration?
Give six clinical signs of shock in infants Explain Isonatraemic and hyponatraemic
dehydration and which people it’s easily occur in?
What happens in hypernatraemic dehydration

A

Assessment
Clinical assessment of dehydration is important but difficult. The most accurate measure of dehydration is the degree of weight loss during the diarrhoeal illness. A recent weight measurement is useful but is often not available and may be misleading if the child had clothes on or the different measuring scales are not accurate. The history and examination are used to assess the degree of dehydration as:
• noclinicallydetectabledehydration(usually<5% loss of body weight)
• clinicaldehydration(usually5%to10%lossof body weight)
• shock(usually>10%lossofbodyweight;Fig.14.9 and Table 14.1). Shock must be identified without delay.

Decreased level of consciousness,Sunken fontanelle,Dry mucous membranes,Eyes sunken and tearless,Tachypnoea,Prolonged capillary refill time, Pale or mottled skin
Hypotension, Tachycardia ,Weak peripheral pulses,Reduced tissue turgor,Sudden weight loss,Reduced urine output, cold extremities

Isonatraemic and hyponatraemic
dehydration
In dehydration, there is a total body deficit of sodium and water. In most instances, the losses of sodium and water are proportional and plasma sodium remains within the normal range (isonatraemic dehydration). When children with diarrhoea drink large quantities of water or other hypotonic solutions, there is a greater net loss of sodium than water, leading to a fall in plasma sodium (hyponatraemic dehydration). This leads to a shift of water from extracellular to intracellular com- partments. The increase in intracellular volume leads to an increase in brain volume, which may result in seizures, whereas the marked extracellular depletion leads to a greater degree of shock per unit of water loss. This form of dehydration is more common in poorly nourished infants in developing countries.

Hypernatraemic dehydration
Infrequently, water loss exceeds the relative sodium loss and plasma sodium concentration increases (hypernatraemic dehydration). This usually results from high insensible water losses (high fever or hot, dry environment) or from profuse, low-sodium diar- rhoea. The extracellular fluid becomes hypertonic with respect to the intracellular fluid, which leads to a shift of water into the extracellular space from the intracellular compartment. Signs of extracellular fluid depletion are therefore less per unit of fluid loss, and depression of the fontanelle, reduced tissue elasticity, and sunken eyes are less obvious. This makes this form of dehydration more difficult to recognize clinically, particularly in an obese infant. It is a particularly dangerous form of dehydration as water is drawn out of the brain and cerebral shrinkage within a rigid skull may lead to jittery movements, increased muscle tone with hyperreflexia, altered consciousness, seizures, and multiple, small cerebral haemorrhages. Transient hyperglycaemia occurs in some patients with hypernatraemic dehydration; it is self-correcting and does not require insulin.

25
Q

A number of disorders may masquerade as gastroenteritis and, when in doubt, hospital referral is essential. Name these disorders

What investigations are done for dehydration or gastroenteritis
How is it managed(if there’s no dehydration,if there’s dehydration,if there’s shock )

In gastroenteritis, death is from dehydration; its prevention or correction is the mainstay of management
Rapid intravenous therapy is indicated in shock from
gastroenteritis. However, it may be harmful in head injury, malnutrition or diabetic ketoacidosis. It was also harmful in a trial of severe febrile illness without shock in children in Africa (FEAST trial). Under these circumstances, intravenous fluids should be given cautiously and clinical response monitored closely. True or false

A

Systemic infections:Septicaemia, meningitis

Local infections:Respiratory tract infection,
otitis media, hepatitis A, urinary tract infection
Surgical disorders:Pyloric stenosis, intussusception, acute appendicitis, necrotizing enterocolitis, Hirschsprung disease
Metabolic disorders:Diabetic ketoacidosis
Renal disorders:Haemolytic uraemic syndrome
Others
Coeliac disease, cow’s milk protein allergy, lactose intolerance, adrenal insufficiency

Investigation
Usually, no investigations are indicated. Stool culture is required if the child appears septic, if there is blood or mucus in the stools, or the child is immunocom- promised. It may be indicated following recent foreign travel, if the diarrhoea has not improved by day 7, or if the diagnosis is uncertain. Plasma electrolytes, urea, creatinine, and glucose should be checked if intravenous fluids are required or there are features suggestive of hypernatraemia. If antibiotics are started, a blood culture should be taken.

No dehydration:

Prevent dehydration
Continue breastfeeding and other milk feeds
Encourage fluid intake to compensate for increased gastrointestinal losses Discourage fruit juices and carbonated drinks
Oral rehydration solution (ORS) as supplemental fluid if at increased risk of dehydration

Dehydration:

Oral rehydration solution
Give fluid deficit replacement (50 ml/kg) over 4 hours as
well as maintenance fluid requirement. Give ORS often and in small amounts
Continue breastfeeding
Consider supplementing ORS with usual fluids if inadequate intake of ORS
If inadequate fluid intake or vomits persistently, consider giving ORS via nasogastric tube
If there’s
Deterioration or persistent vomiting, do intravenous therapy for rehydration:Replace fluid deficit over 24 hours in most cases and give maintenance fluids Unless a recent weight measurement is available, clinical estimation of hydration status is difficult. Consider fluid deficit to be 100 ml/kg (10% body weight) if shock is present and 50 ml/kg (5% body weight) if not in shock
For maintenance fluids see Table 6.1
Give 0.9% sodium chloride solution or 0.9% sodium chloride solution with
5% glucose
Monitor plasma electrolytes, urea, creatinine, and glucose. Consider intravenous potassium supplementation
Continue breastfeeding if possible

After rehydration :

After rehydration
Give full strength milk and reintroduce usual solid food
Avoid fruit juices and carbonated drinks
Advise parents – diligent hand washing, towels used by infected child not to be shared, do not return to childcare facility or school until 48 hours after last episode

Shock
Intravenous therapy
Give bolus of 0.9% sodium chloride solution. Repeat if necessary. If remains shocked, consider consulting paediatric intensive care specialist. If signs of shock improve give IV therapy for rehydration (same as in dehydration),After this, do things after rehydration (same as in dehydration)

26
Q

Explain hypernatraemic dehydration
The fluid deficit should be replaced over how many hours with what?
What is the mechanism of action of ORS
Why don’t you give antidiarreals and antiemetics in gastroenteritis,when will you give antibiotics,and why is nutrition important

A

Hypernatraemic dehydration
The management of hypernatraemic dehydration can be particularly difficult. Oral rehydration solution should be used to rehydrate hypernatraemic children with clinical dehydration. If intravenous fluids are required, a rapid reduction in plasma sodium concentration and osmolality will lead to a shift of water into cerebral cells and may result in seizures and cerebral oedema. The reduction in plasma sodium should therefore be slow. The fluid deficit should be replaced over at least 48 hours (with 0.9% or 0.45% saline) and the plasma sodium measured regularly, aiming to reduce it at less than 0.5 mmol/l per hour.

he mechanism of action of oral rehydration solu- tion is shown in Fig. 14.11. Large quantities of sodium are excreted into the intestine, but nearly all is reab- sorbed. The primary mechanism of sodium absorption is by a glucose–sodium transporter, with the active absorption of sodium allied to the absorption of glucose. The sodium is then actively pumped from epi- thelial cells into the circulation via sodium/potassium adenosine triphosphatase, creating an electrochemical gradient that water moves down. A second mechanism is via an active, linked sodium–hydrogen exchanger.
If an oral solution contains both sodium and glucose, sodium and passive water absorption is increased. This works effectively even in the presence of inflammation of the gut, and is therefore effective in diarrhoeal illness. The oral rehydration solution does not ‘stop’ the diarrhoea, which often continues, but the absorp- tion of water and solutes exceeds secretion and keeps the child hydrated until the infective organism is eradicated. Coca-Cola and apple juice have a much lower sodium content and higher osmolarity than oral rehydration solution and are unsuitable as oral rehy- dration solutions.

Antidiarrhoeal drugs (e.g. loperamide,
Lomotil) and antiemetics
There is no place for medications for the vomiting or diarrhoea of gastroenteritis in children as they:
• areineffective
• mayprolongtheexcretionofbacteriainstools • canbeassociatedwithside-effects
• addunnecessarilytocost
• focusattentionawayfromoralrehydration.

Antibiotics
Antibiotics are not routinely required for gastroenteritis even if there is a bacterial cause. They are only indicated for suspected or confirmed sepsis, extra- intestinal spread of bacterial infection, for salmonella gastroenteritis if aged under 6 months, in malnourished or immunocompromised children, or for specific bac- terial or protozoal infections (e.g. Clostridium difficile associated with pseudomembranous colitis, cholera, shigellosis, giardiasis).
Nutrition
In developing countries, multiple episodes of diarrhoea are a major contributing factor to the development of malnutrition. Following diarrhoea, nutritional intake should be increased. Diarrhoea may be associated with zinc deficiency and supplementation may be helpful in both acute diarrhoea and as prophylaxis.

27
Q

What is post gastroenteritis syndrome,in summary of gastroenteritis(results in death from what? It’s mostly of what cause but can be caused by which other organisms, which people are particularly susceptible to dehydration, how is dehydration assessed? ,what is the mainstay of treatment? When is IV fluid required?)
How is malabsorption manifested as?
What are the Characteristics of true malabsorption stool? What is it inappropriate to do ? Which disorders may cause malabsorption of many nutrients? And some disorders are highly specific give an example

A

Postgastroenteritis syndrome
Infrequently, following an episode of gastroenteritis, the introduction of a normal diet results in a return of watery diarrhoea. In such cases, oral rehydration therapy should be restarted.

Gastroenteritis
• Results in death from dehydration of hundreds of thousands of children worldwide every year.
• Is mostly viral, but it can be caused by Campylobacter, Shigella, and Salmonella and other organisms.
• Infants are particularly susceptible to dehydration.
• Dehydration is assessed as no clinical dehydration, clinical dehydration or shock according to symptoms and signs, but clinical assessment of severity is problematic.
• Oral rehydration solution is the mainstay of treatment and usually effective; intravenous fluid is only required for shock or ongoing vomiting or clinical deterioration.

Malabsorption
Disorders affecting the digestion or absorption of nutrients manifest as:
• abnormalstools
•poor weight gain or faltering growth in most but
not all cases
• specificnutrientdeficiencies,eithersinglyorin
combination.

The true malabsorption stool is difficult to flush down the toilet and has an odour that pervades the whole house. In general, colour is a poor guide to abnormality. Reliable dietetic assessment is important. It is inappropriate to investigate children for malabsorption as a cause of their faltering growth when dietary energy intake is demonstrably low and other symptoms are absent. Some disorders affect- ing the small intestinal mucosa or pancreas (chronic pancreatic insufficiency) may lead to the malabsorp- tion of many nutrients (pan-malabsorption), whereas others are highly specific, e.g. zinc malabsorption in acrodermatitis enteropathica

28
Q

What is coeliac disease ? What does it cause? The age at presentation is partly influenced by what? What’s the classical presentation? However, this ‘classical’ form is no longer the most common presentation and children are now more likely to present less acutely in later childhood. The clinical features of coeliac disease can be highly variable and include what? When can it be identified? How is coeliac disease diagnosed? Name two serological tests used . How is it managed? What happens when they don’t follow it

In summary ,what is coeliac disease,when is the classical presentation at what age with what signs? Name some more subtle modes of presentation?
How is it diagnosed? How is it treated?

A

Coeliac disease
Coeliac disease is an enteropathy in which the gliadin fraction of gluten and other related prolamines in wheat, barley, and rye provoke a damaging immu- nological response in the proximal small intestinal mucosa. As a result, the rate of migration of absorptive cells moving up the villi (enterocytes) from the crypts is massively increased but is insufficient to compensate for increased cell loss from the villous tips. Villi become progressively shorter and then absent, leaving a flat mucosa.
The age at presentation is partly influenced by the age of introduction of gluten into the diet.
The classical presentation is of a profound malab- sorptive syndrome at 8–24 months of age after the introduction of wheat-containing weaning foods. There is faltering growth, abdominal distension and buttock wasting, abnormal stools, and general irrita- bility.The clinical features of coeliac disease can be highly variable and include mild, non-specific gastrointestinal symptoms, anaemia (iron and/or folate deficiency) and growth faltering. Alternatively, it is identified on screening of children at increased risk (type 1 diabetes mellitus, autoimmune thyroid disease, Down syndrome) and first-degree relatives of individu- als with known coeliac disease.

Diagnosis
Although the diagnosis is strongly suggested by posi- tive serology, confirmation depends upon the demon- stration of mucosal changes (increased intraepithelial lymphocytes and a variable degree of villous atrophy and crypt hypertrophy) on small intestinal biopsy performed endoscopically followed by the resolu- tion of symptoms and catch-up growth upon gluten withdrawal.
Although there is no place for the empirical use of a gluten-free diet as a diagnostic test for coeliac disease, strongly positive serological tests such as anti-tTG and EMA in symptomatic individuals may make the need for biopsy confirmation unnecessary in a small propor- tion of children following assessment by a paediatric gastroenterologist.

Management
All products containing wheat, rye, and barley are removed from the diet and this results in resolution of symptoms. Supervision by a dietician is essential. In children in whom the initial biopsy or the response to gluten withdrawal is doubtful, a gluten challenge may be required in later childhood to demonstrate continuing susceptibility of the small intestinal mucosa to damage by gluten. The gluten-free diet should be adhered to for life. Non-adherence to the diet risks the development of micronutrient deficiency, especially osteopenia, and there is a small but definite increased risk in bowel malignancy, especially small bowel lymphoma.

Coeliac disease
• A gluten-sensitive enteropathy.
• Classical presentation is at 8–24 months of age
with abnormal stools, faltering growth, abdominal distension, muscle wasting, and irritability (now rare).
• Other, more subtle, modes of presentation – e.g. short stature, anaemia, abdominal pain and screening, e.g. children with diabetes mellitus, are now much more common

Diagnosis – positive serology (immunoglobulin A tissue transglutaminase and endomysial antibodies), flat mucosa on duodenal biopsy (usually but not always performed), and resolution of symptoms and catch-up growth upon gluten withdrawal.
• Treatment – gluten-free diet for life.

29
Q

In which people will short bowel syndrome occur? Why will it occur? What does children getting malabsorption diarrhea and malnutrition depend on in nutrient malabsorption. If the malabsorption diarrhea and things are severe what may the kids require? These kids are at risk of what diseases?
State some causes of nutrient malabsorption

A

Short bowel syndrome usually occurs when an infant
or child has had a large surgical resection. This may be due to congenital atresia, necrotizing enterocolitis, malrotation with volvulus, or a traumatic event such as a road traffic accident. Depending on the length and type of residual bowel (ileum or jejunum) and pres- ence of ileocaecal valve, these children may develop malabsorption diarrhoea and malnutrition. If severe, they may require supplemental parenteral nutrition for their intestinal failure. They are at risk from nutritional deficiencies, intestinal failure-associated liver disease, and central line-associated bloodstream infections, and need to be managed by specialist multidisciplinary teams.

Cholestatic liver disease or biliary atresia:
Bile salts no longer enter duodenum in the bile. This leads to defective solubilization of
the products of triglyceride hydrolysis. Fat and fat-soluble malabsorption result
Lymphatic leakage or obstruction:
Chylomicrons (containing absorbed lipids) unable to reach thoracic duct and the systemic circulation, e.g. intestinal lymphangiectasia
(abnormal lymphatics)
Short bowel syndrome:
Small-intestinal resection, due to congenital anomalies or necrotizing enterocolitis, leads to nutrient, water and electrolyte malabsorption
Loss of terminal ileal function:
e.g. resection or Crohn’s disease Absent bile acid and vitamin B12 absorption

Exocrine pancreatic dysfunction, e.g. cystic fibrosis
Absent lipase, proteases, and amylase lead to defective digestion of triglyceride, protein, and starch (’pan-nutrient malabsorption’)
Small-intestinal mucosal disease
• Loss of absorptive surface area, e.g. coeliac disease
• Specific enzyme defects, e.g. transient lactase deficiency following gastroenteritis, but is uncommon
• Specific transport defects, e.g. glucose–galactose malabsorption (severe life- threatening diarrhoea with first milk feed), acrodermatitis enteropathica (zinc malabsorption, also erythematous rash around mouth and anus)

30
Q

What is chronic non specific diarrhoea? What’s another name for it?
What are the characteristics of the stools? What kind of stools are common? In a proportion of kids the diarrhoea may be due to what? Occasionally what’s the cause of chronic non specific diarrhea? Once possible underlying causes have been excluded, in the majority of cases the loose stools probably result from what? In summary,in an infant with faltering growth in chronic diarrhea,consider what two things?
When will you consider malabsorption? When will you consider chronic non specific diarrhea?
In inflammatory bowel disease which disease is more common than ulcerative colitis? What’s the difference between these two? What can inflammatory bowel disease cause?

A

Chronic non-specific diarrhoea
This condition, previously known as toddler diarrhoea, is the most common cause of persistent loose stools
in preschool children. Characteristically, the stools are of varying consistency, sometimes well formed, some- times explosive and loose. The presence of undigested vegetables in the stools is common. Affected children are well and thriving. In a proportion of children the diarrhoea may result from undiagnosed coeliac disease or excessive ingestion of fruit juice, especially apple juice. Occasionally the cause is temporary cow’s milk allergy following gastroenteritis, when a trial of a cow’s milk protein free diet may be helpful. Once possible underlying causes have been excluded, in the majority of cases the loose stools probably result from dysmo- tility of the gut (a form of irritable bowel syndrome) and fast-transit diarrhoea; it almost always improves with age.

Summary
Chronic diarrhoea
• In an infant with faltering growth, consider coeliac disease and cow’s milk protein allergy.
• Following bowel resection, cholestatic liver disease or exocrine pancreatic dysfunction, consider malabsorption.
• In an otherwise well toddler with undigested
vegetables in the stool, consider chronic
non-specific diarrhoea

Inflammatory bowel disease
Approximately a quarter of patients present in child- hood or adolescence, and in contrast with the adult population, Crohn’s disease is more common than ulcerative colitis. Crohn’s disease can affect any part of the gastrointestinal tract from mouth to anus, whereas in ulcerative colitis the inflammation is confined to the colon. Inflammatory bowel disease may cause poor general health, restrict growth, and have an adverse effect on psychological well-being. Management requires a specialist multidisciplinary team.

31
Q

What are the clinical features of Crohn’s disease in kids and adolescents? What kind of symptoms can be seen in older kids? Why May there be considerable delay in diagnosis? What disease can it mimic?
What signs are helpful in making a diagnosis and confirming a relapse?
What is Crohn’s disease
Diagnosis is based on what ? What is the histological hallmark of Crohn’s disease? Smal bowl imaging may reveal what? What is the treatment

A

Crohn’s disease

Growth failure Puberty delayed
Classical presentation (25%): • abdominal pain
• diarrhoea
• weight loss General ill health: • fever
• lethargy
• weight loss
Extra-intestinal manifestations:
• oral lesions
or perianal skin tags
• uveitis
• arthralgia
• erythema nodosum

Lethargy and general ill health without gastrointestinal symptoms can be the presenting features, particularly in older children. There may be considerable delay in diagnosis as it may be mistaken for psychological problems. It may also mimic anorexia nervosa. The presence of raised inflammatory markers (platelet count, erythrocyte sedimentation rate, and C-reactive protein), iron-deficiency anaemia, and low
serum albumin are helpful in both making a diagnosis and confirming a relapse. Crohn’s disease is a transmural, focal, subacute, or chronic inflammatory disease, most commonly affecting the distal ileum and proximal colon
. Initially, there are areas of acutely inflamed, thickened bowel. Subsequently, strictures of the bowel and fistulae may develop between adjacent loops of bowel, between bowel and skin or to other organs (e.g. vagina, bladder).
Diagnosis is based on endoscopic and histological findings on biopsy. Upper gastrointestinal endo- scopy, ileocolonoscopy and small bowel imaging are required. The histological hallmark is the presence of non-caseating epithelioid cell granulomata, although this is not identified in up to 30% at presentation. Small bowel imaging may reveal narrowing, fissuring, mucosal irregularities and bowel wall thickening.
Remission is induced with nutritional therapy, when the normal diet is replaced by whole protein modular feeds (polymeric diet) for 6–8 weeks. This is effective in 75% of cases. Systemic steroids are required if ineffective.
Relapse is common and immunosuppressant medi- cation (azathioprine, mercaptopurine or methotrexate) is almost always required to maintain remission. Anti- tumour necrosis factor agents (infliximab or adalimu- mab) may be needed when conventional treatments have failed. Long-term supplemental enteral nutrition (often with overnight nasogastric or gastrostomy feeds) may be helpful in correcting growth failure. Surgery is necessary for complications of Crohn’s disease – obstruction, fistulae, abscess formation or severe localized disease unresponsive to medical treat- ment, often manifesting as growth failure. In general, the long-term prognosis for Crohn’s disease beginning in childhood is good and most patients lead normal lives, despite occasional relapsing disease.
Growth failure and delayed puberty are features of Crohn’s disease in children.

32
Q

What is ulcerstive colitis? What do the patients present w characteristically?
Which two signs are less frequent in Crohn’s disease? Extra intestinal complci include which diseases? How is diagnosis made? What are the endoscopy features in kids and what’s the features in adults. What are the histological features
What is the essence of small bowel imaging? In mild disease What is the treatment for ulcerative colitis? Disease confined to the rectum and sigmoid may be managed w what? More aggressive disease requires what?

Severe fulminating disease is a medical emergency and requires treatment with intravenous fluids and steroids. If this fails to induce remission, ciclosporin may be used.
Colectomy with an ileostomy or ileorectal pouch is undertaken for severe fulminating disease, which may be complicated by a toxic megacolon, or for chronic poorly controlled disease. True or false why should regular colonoscopic screening be performed after ten years from diagnosis?
How does hirschsprung disease present? Why does it occur? What two signs later develop? What does rectal exam reveal? What can lead to delayed diagnosis? Occasionally what can infants present w during first few weeks of life? In later childhood the presentation is with what? How is diagnosis made? How is it managed ?

A

Ulcerative colitis
Ulcerative colitis is a recurrent, inflammatory and ulcerating disease involving the mucosa of the colon. Characteristically, the disease presents with rectal bleeding, diarrhoea and colicky pain. Weight loss and growth failure may occur, although this is less frequent than in Crohn’s disease. Extra-intestinal complications include erythema nodosum and arthritis.
The diagnosis is made on endoscopy (upper and ileocolonoscopy) and on the histological features, after exclusion of infective causes of colitis. There is a confluent colitis extending from the rectum proximally for a variable length. In contrast to adults, in whom the colitis is usually confined to the distal colon, 90% of children have pancolitis. Histology reveals mucosal inflammation, crypt damage (cryptitis, architectural distortion, abscesses and crypt loss), and ulceration. Small bowel imaging is required to check that extra- colonic inflammation suggestive of Crohn’s disease is not present.
In mild disease, aminosalicylates (e.g. mesalazine) are used for induction and maintenance therapy. Disease confined to the rectum and sigmoid colon (rare in children) may be managed with topical steroids. More aggressive or extensive disease requires systemic ster- oids for acute exacerbations and immunomodulatory therapy, e.g. azathioprine alone to maintain remission or in combination with low-dose corticosteroid therapy. There is a role for biological therapies such as infliximab or ciclosporin in patients with resistant disease, but if ineffective, surgery should not be delayed.
There is an increased inci- dence of adenocarcinoma of the colon in adults (1 in 200 risk for each year of disease between 10–20 years from diagnosis). Regular colonoscopic screening is performed after 10 years from diagnosis.

Hirschsprung disease
The absence of ganglion cells from the myenteric and submucosal plexuses of part of the large bowel results in a narrow, contracted segment. The abnormal bowel extends from the rectum for a variable distance proximally, ending in a normally innervated, dilated colon. In 75% of cases, the lesion is confined to the rectosigmoid, but in 10% the entire colon is involved. Presentation is usually in the neonatal period with intestinal obstruction heralded by failure to pass meconium within the first 24 hours of life. Abdominal distension and later bile-stained vomiting develop (Fig. 14.16). Rectal examination may reveal a narrowed segment and withdrawal of the examining finger often releases a gush of liquid stool and flatus. Temporary improvement in the obstruction following the dilata- tion caused by the rectal examination can lead to a delay in diagnosis. Occasionally, infants present with severe, life- threatening Hirschsprung enterocolitis during the first few weeks of life. In later childhood, presentation is with chronic constipation, usually profound, and asso- ciated with abdominal distension but usually without soiling. Growth failure may also be present.
Diagnosis is made by demonstrating the absence of ganglion cells, together with the presence of large, acetylcholinesterase-positive nerve trunks on a suction rectal biopsy. Anorectal manometry or barium studies may be useful in giving the surgeon an idea of the length of the aganglionic segment but are unreliable for diagnostic purposes. Management is surgical and usually involves an initial colostomy followed by anas- tomozing normally innervated bowel to the anus.

Summary
Hirschsprung disease
• Absence of myenteric plexuses of rectum and variable distance of colon.
• Presentation – usually intestinal obstruction in the newborn period following delay in passing meconium. In later childhood – profound chronic constipation, abdominal distension, and growth failure.
• Diagnosis – suction rectal biopsy.

33
Q

Parents may use constipation to describe what?
Infants have an average ego how many stools per day in the first week of life and how many stools per day by 1 year? What about for breastfed infants? What is constipation? What signs may be with the constipation?
The constipatiok may be precipitated by what? In older kids it may relate to what problems? Examina usually reveals what? Name some causes of constipation.
What are the red flag signs and symptoms indicative of a more complex problem causing the constipation? (State the sign and the disease it shows) In these causes what exam shouldn’t be performed ? Which people will perform that exam and why? Investigations are not usually required to diagnose idiopathic con- stipation, but are carried out as indicated by history or clinical findings. True or false

Constipation arising acutely in young children, e.g. after an acute febrile illness, usually resolves spontane- ously or with the use of maintenance laxative therapy and extra fluids. True or false

A

Constipation is an extremely common reason for consultation in children. Parents may use the term to describe decreased frequency of defecation, the degree of hardness of the stool or painful defecation. The ‘normal’ frequency of defecation is highly variable and varies with age. Infants have an average of four stools per day in the 1st week of life, but this falls to an average of two per day by 1 year of age. Breastfed infants may not pass stools for several days and be
entirely healthy. After 1 year of age, most children have a daily bowel action.
A pragmatic definition of constipa- tion is the infrequent passage of dry, hardened faeces often accompanied by straining or pain and bleeding associated with hard stools. There may be abdominal pain, which waxes and wanes with passage of stool or overflow soiling.The constipation may have been pre- cipitated by dehydration or reduced fluid intake or an anal fissure causing pain. In older children, it may relate to problems with toilet training, refusal and anxieties about opening bowels at school or in unfamiliar toilets.
Examination usually reveals a well child whose growth is normal, the abdomen is soft and any abdominal distension is normal for age. The back and perianal area are normal in appearance and position. A soft faecal mass may sometimes be palpable in the lower abdomen, but is not necessary for the diagnosis.
A primary underlying cause for constipation is rare, but a number of underlying conditions should be consid- ered: Hirschsprung disease, lower spinal cord prob- lems, anorectal abnormalities, hypothyroidism, coeliac disease and hypercalcaemia.

Failure to pass meconium within 24 hours of life :Hirschsprung disease Faltering growth/growth failure :Hypothyroidism, coeliac disease, other causes
Gross abdominal distension:Hirschsprung disease or other gastrointestinal dysmotility Abnormal lower limb neurology or deformity, e.g. talipes or secondary urinary incontinence :Lumbosacral pathology
Sacral dimple above natal cleft, over the spine – naevus, hairy patch, central pit, or discoloured skin :Spina bifida occulta
Abnormal appearance/ position/patency of anus :Abnormal anorectal anatomy
Perianal bruising or multiple fissures :Sexual abuse
Perianal fistulae, abscesses, or fissures : Perianal Crohn’s disease

Digital rectal examination should not be performed; it may sometimes be considered by a paediatric specialist to help identify anatomical abnormalities or Hirschsprung disease.

34
Q

What happens in more long standing constipation? Why will involuntary soiling occur? Management of these children is likely to be more difficult and pro- tracted and often requires a multidisciplinary approach .Children of school age are frequently teased as a result and secondary behavioural problems are common.
It should be explained to the child and the parents that the soiling is involuntary and that recovery of normal rectal size and sensation can be achieved but may take a long time. True or false. What is the jntial sim of management for this child? How can it be generally achieved? State the dose regimen. If it doesn’t work what drug will you move on to? If the polyethylene glycol + electrolytes is not tolerated what can be given in exchange? What must be followed by maintenance treatment and why? What drug is the treatment of choice for this ? Why isn’t addition of fibers to diet useful? What should the child be encouraged to do after mealtimes?
What is a common cause for treatment failure? Dietary interventions alone are of little or no benefit in managing constipation in this situation, although the child should receive sufficient fluid and a balanced diet. True or false

A

In more long-standing constipation, the rectum becomes overdistended, with a subsequent loss of feeling the need to defecate. Involuntary soiling may occur as contractions of the full rectum inhibit the internal sphincter, leading to overflow. The initial aim is to evacuate the overloaded rectum completely. This can generally be achieved using a disimpaction regimen of stool softeners, initially with a macrogol laxative, e.g. polyethylene glycol 3350 + electrolytes (Movicol Paediatric Plain). An escalating dose regimen is administered over 1–2 weeks or until impaction resolves. If this proves unsuccessful, a stimulant laxa- tive, e.g. senna, or sodium picosulphate, may also be required. If the polyethylene glycol + electrolytes is not tolerated, an osmotic laxative (e.g. lactulose) can be substituted.
Disimpaction must be followed by maintenance treatment to ensure ongoing regular, pain-free defeca- tion. Polyethylene glycol (with or without a stimulant laxative) is generally the treatment of choice. The dose should be gradually reduced over a period of months in response to improvement in stool consistency and frequency.
The addition of extra fibre to the diet is not helpful, and may make stools larger and more difficult to pass. The child should be encour- aged to sit on the toilet after mealtimes to utilize the
physiological gastrocolic reflex and improve the likeli- hood of success.
The outcome is more likely to be successful if the child is engaged in the treatment process. This requires exploring the child’s concerns and motivation to change. Sometimes use of behavioural interven- tions, e.g. a star chart, is helpful to record and reward progress, as well as to motivate the child.
Encouragement by family and health professionals is essential, as relapse is common and psychological support is sometimes required. The mainstay of treat- ment is the early, aggressive and prolonged use of laxative medication in a dose that allows the passage of a large, soft stool at least once a day. One needs to emphasise that the use of laxatives is safe, even long-term, as underuse is the commonest reason for treatment failure. Occasionally, the faecal retention is so severe that evacuation is only possible using enemas or by manual evacuation under an anaesthetic. They should only be performed under specialist supervision, paying particular attention to avoiding distress and embarrassment for the child.

35
Q

Under genital disorders in kids what are the features?
For inguinoscrotal conditions ,how is development of a testis from an early indeterminate gonad determined? What will cause a testis to descend ?
What structures are found in the scrotum in a boy? What makes up coverings of the spermatic cord in a boy? What will cause fluid or abdominal contents to become a hydrocele or hernia respectively?
What is a variocele? What causes it?
Which side of the scrotum does it occur in more commonly and why? What will you see on examination? How is it managed?

A

Features of genital disorders in children are:
• hydroceles,inguinalherniasandundescended testes usually arise from abnormal embryological development
• theacutescrotumisasurgicalemergency
• foreskinconditionsandhypospadiasarecommon
in boys
• vulvovaginitis and labial adhesions are common in
girls.

Inguinoscrotal conditions
Embryology
Development of a testis from an early indeterminate gonad is determined by genes associated with a Y chro- mosome. For a testis to descend from its origin on the posterior abdominal wall, it must produce testosterone which acts on peripheral tissues. The testis, guided by the mesenchymal gubernaculum, migrates down into the inguinal canal .The structures that are found in the scrotum in a boy (testis, vas and blood vessels) or labium in a girl (attachment of the round ligament of the uterus) pass through the abdominal wall and pick up layers corresponding to those of the abdominal wall. In a boy these make up the cover- ings of the spermatic cord. In boys and girls there is a remnant of peritoneal invagination, the processus vaginalis (Fig. 20.1b), which, if it remains patent and in continuity with the abdomen, explains why fluid or abdominal contents can become a hydrocele or hernia, respectively

Varicocele
This is a scrotal swelling comprising dilated (varicose) testicular veins and occurs in up to 15% of boys, usually at puberty (Fig. 20.4). Its cause is multifactorial; valvular incompetence plays a role. It is commoner on the left side because of drainage of the gonadal vein into the left renal vein, which also receives blood containing catecholamines from the left adrenal vein. It is usually asymptomatic, but may cause a dull ache. On examina- tion it may have a bluish colour and feel like a ‘bag of
worms’. Sometimes the testis is smaller or softer than normal. Management is conservative if asymptomatic. Occlusion of the gonadal veins can be achieved by surgical ligation – through the groin laparoscopically or by radiological embolization.

36
Q

Inguinak hernias are common in which kids? What causes a hernia? Hernia is usually of what type and occuring in which people. The other type which is less common is more likely in which people? How does a hernia present?

They are usually asymptomatic but may be intermittent, visible during what? On exam what will you see? In what circumstances will the lump be tender and the infant be irritable and vomit? The risk of incarceration is much higher in which people? Most hernias can be successfully reduced by what? When can surgery be planned? Why is an emergency surgery required when reduction is impossible? What can happen to girls sometimes when this happens? What does the surgery involve? At what age can this surgery be performed as a day case? Prompt surgical repair is indicated for inguinak hernias in which people? What is hydrocele? How do they usually appear? It is usually possible to feel what however tense the hydrocele ? How do you differentiate between hernia and a hydrocele? Why can hydroceles be resolved spontaneously usually? When will surgery be required? In a girl, a hydrocele (of the ‘canal of Nuck’) is much less common than in boys. True or false

A

Inguinal hernia
Inguinal hernias are common, occurring in up to 5% of boys, and are even more common in premature babies. As explained by the embryology, a hernia is usually caused by a persistently patent processus vaginalis and emerges from the deep inguinal ring through the
inguinal canal. It is therefore usually indirect, although in premature babies where the tissues are weak and friable, direct hernias are more likely than in older children.
A hernia presents as a lump in the groin which may extend into the scrotum or labium. They are usually asymptomatic but may be intermittent, visible during straining. On examination, sometimes a lump or thickened cord structures can be palpated in the groin.
The contents of the hernia may become irreducible (incarcerated), causing pain and sometimes intestinal obstruction or damage to the testis (strangulation). In these circumstances the lump is tender and the infant may be irritable and may vomit. The risk of incarcera- tion is much higher in infants than in older children.
Most hernias can be successfully reduced by ‘taxis’ (gentle compression in the line of the inguinal canal) with good analgesia. Surgery can then be planned for a suitable time when any oedema has settled and the child is well. If reduction is impossible, emergency surgery is required because of the risk of compromise of the bowel or testis. In girls, sometimes the ovary can become incarcerated within a hernia.
Surgery involves ligation and divi- sion of the processus vaginalis, which has become the hernial sac (herniotomy, removal of the hernia sac – as opposed to herniorrhaphy in adults, when the inguinal abdominal wall is also reinforced, usually with a mesh). Beyond the first three months of age, this can be safely performed as a day case.
Prompt surgical repair is indicated for inguinal hernias in infants to lower the risk of incarceration
Hydrocele
A hydrocele has the same underlying anatomy as a hernia, but the processus vaginalis, although patent, is not sufficiently wide to form an inguinal hernia. Hydroceles are usually asymptomatic and some- times appear blue. It is usually possible to feel the testis, however tense the hydrocele. Sometimes the hydrocele is separate from the testis in the cord. The key to differentiating a hernia from a hydrocele is the ability to ‘get above’ a hydrocele. Hydroceles usually transilluminate (Fig. 20.3).
Although the processus vaginalis is often patent at birth it usually closes within months. Hydroceles therefore usually resolve spontaneously, and can be managed expectantly. Surgery may be considered if it persists beyond the first two years of life, but resolu- tion may take longer than this.

37
Q

Most undescended testes become arrested along their normal pathway of descent true or false? Undescended tested is more common in which groups of kids? When should the diagnosis be ideally made? Which environment should exam of the testes in babies be done? How is th exam done? On exam an undescended testis may be what or what? How will you know if it’s palpable? Occasionally where can it be palpated? What happens when the testis is impalpable?
What do you do when there are bilateral impalpable testes?
What is the difference between a retractile and and undescended testis?
What muscle pulls up the testis?
why should a boy with a suspected undescended testis be examined in a warm environment and when warm and relaxed.

A

Undescended testis
.Unde- scended testes are present in up to 5% of newborn term infants but are more common in premature infants. By three months of age, only 1% are still unde- scended. The diagnosis should ideally be made at the routine examination of the newborn but since there is still a small spontaneous rate of descent after this time the decision to operate for undescended testis should be delayed.
Examination of the testes in babies must be made in a warm environment and with warm hands. The testes may be felt in the scrotum or may need to be delivered by gentle pressure along the line of the inguinal canal to the scrotum.
An undescended testis may be palpable or impal- pable. A palpable undescended testis is usually seen or felt in the groin, but cannot be manipulated into the scrotum. Occasionally it can be palpated below the external inguinal ring but outside the scrotum – the so-called ‘ectopic’ testis.
If the testis is impalpable, it may be in the inguinal canal but cannot be identified or it may be intra- abdominal or absent. If there are bilateral impalpable testes, the karyotype must be established to exclude disorders of sex development. This should be regarded as a medical emergency.
A testis may also be retractile. The crucial difference between a retractile and undescended testis is that a retractile testis can be manipulated into the scrotum with ease and without tension. Action of the cremaster muscle (as seen in eliciting the cremasteric reflex by light touch on the abdominal wall) pulls up the testis. Parents of boys with a retractile testis often report that the testis is sometimes obvious, particularly when the boy is warm and relaxed, and sometimes not.

38
Q

Why is orchidopexy performed? When should it be performed? Groin approach orchidopexy involves what? How is an intra-abdominal testis is usually managed? Regarding impalpable testes, about 10% have regressed in development and are, in fact, absent. Laparoscopy allows both diagnosis and treatment. True or false
Why is follow up recommended for a retractile testis? Undescended testes should be referred to a paediatric surgeon when detected. If surgery is required, the optimum time is within the first year of life. True or false
In the acute scrotum,what condition is commonest in post pubertal boys ? This condition is very painful with what other signs? The pain may be localized to which area? What need does it highlight? How is this condition distinguished from an incarcerated hernia? This condition must be treated within how many hours of onset? Why is fixation of the contralateral testis essential ? In what condition is testicular loss inevitable

A

Investigations and management
Imaging is not helpful in the assessment of an un- descended testis.
Orchidopexy, the surgical placement of the testis in the scrotum, is performed for the following reasons:
• Cosmetic–toachievethesame,symmetrical appearance as other boys. This may be of psychological benefit. If the testis is absent, a prosthesis can be inserted when older.
• Reducedriskoftorsionandtraumacomparedto groin location
• Fertility–thetestisneedstobeinthescrotum, below body temperature, in order to allow spermatogenesis. The effect is probably marginal in unilateral undescended testis but is more important if bilateral. There is some evidence that delaying orchidopexy beyond the first two years of life adversely affects testicular development.
• Malignancy–increasedriskinanundescended testis, which is greater if bilateral or intra- abdominal. Placing the testis in the scrotum facilitates self-examination but may not influence the risk of malignancy.

The timing of orchidopexy depends on local surgical and anaesthetic facilities, but should be per- formed before or around one year of age. Groin approach orchidopexy involves opening the inguinal canal in a similar manner to herniotomy, mobi- lizing the testis whilst preserving the vas and blood vessels and placing it within the scrotum. It is usually performed as a day case. An intra-abdominal testis is usually managed laparoscopically; it may be amenable to placement in the scrotum in a single operation or may require a staged approach.

For a retractile testis, follow up is recommended because some high testes require surgery to place them in the scrotum. Whether or not this is true ascent of the testis is controversial.

Acute inguinoscrotal conditions (‘the acute scrotum’)
Torsion of the testis
This is commonest in post-pubertal boys but may occur at any age, including the newborn when it usually presents at birth and is believed to be perinatal. It is usually very painful, with redness and oedema of the scrotal skin. However, the pain may be localised to the groin or lower abdomen, highlighting the need to always examine the testes in a boy present- ing with sudden-onset pain in the groin, abdomen or scrotum. It must be distinguished from an incarcerated hernia. An undescended testis is at increased risk of torsion, as is a testis lying transversely on its attach- ment to the spermatic cord (the so-called ‘clapper bell’ testis). Torsion of the testis must be treated within hours of the onset of symptoms to lower the risk of testicular 370 loss. In fact, surgical exploration in any acute scrotal presentation is mandatory unless torsion can be excluded with certainty (see below). Fixation of the contralateral testis is essential because of the increased risk of a contralateral torsion, especially if an anatomical abnormality is present in the torted testis. Outcome is variable, depending on time to correction. If delayed, testicular loss is likely. In perinatal testicular torsion, testicular loss is almost inevitable.

39
Q

What is a testicular appendage?
Torsion of the appendix testis which is a condition in acute scrotum tends to affect which people and is more common than torsion of the testis?
Is the pain sudden? Why is scrotal exploration and excision necessary?
When will surgery not be necessary?

Name some other conditions that can cause acute scrotum. Epididymo- orchitis is commoner in which people and more likely with a pre existing what? Why is scrotal exploration necessary in this condition? What may differentiate epididymitis from torsion of the testis? When shouldn’t you delay scrotal exploration. What investigations should be done if there’s an infection causing the acute scrotum? What are the signs of idiopathic scrotal oedema ? Signs of incarcerated hernia causing acute scrotum include? What is an uncommon cause of testicular damage? Recurrent scrotal pain in boys can be difficult to manage. Any associated symptoms or signs such as swelling or redness should be regarded as what condition? Sometimes pro- phylactic fixation is required to exclude intermittent torsion as a cause for recurrent pain. In adulthood, chronic scrotal pain can follow scrotal surgery
Torsion of the testis must be excluded (by emergency exploration if necessary) in boys with an ‘acute scrotum’. Delay leads to testicular loss true or false .

A

Torsion of appendix testis
A testicular appendage (Hydatid of Morgagni) is a Mul- lerian (paramesonephric) remnant usually located on the upper pole of the testis. Torsion of the appendix testis (Fig. 20.5b) tends to affect prepubertal boys and is more common than torsion of the testis. Pain evolves over days, but is not as dramatic as in testicular torsion. Scrotal exploration and excision of the appendage is often necessary because it cannot be differentiated reliably from torsion of the testis. If a ‘blue dot’ can be seen through the scrotal skin and pain is controlled with analgesia, surgery may not be necessary.

Other acute inguinoscrotal conditions
Infection may cause an acute scrotum. Epididymo- orchitis is commoner in infants and small children, and more likely with a pre-existing urological or anorectal malformation. As it may be indistinguish- able from torsion, scrotal exploration may be necessary. Doppler ultrasound of flow pattern in the testicular blood vessels may allow differentiation of epididymitis from torsion of the testis, but must not delay surgical exploration if torsion remains a possibility. A urine sample should be obtained to identify an associated urinary tract infection. Pus should be sent at operation for microbiology to characterize the nature of the infec- tion, but infection may be bacterial or viral. Antibiotics are started empirically.
In idiopathic scrotal oedema there is redness and swelling extending beyond the scrotum into the thigh, perineum and suprapubic area, but the testis is normal and non-tender. It requires analgesia and review. It may recur. An incarcerated hernia may also cause an acute scrotum, although symptoms usually affect the groin.
Trauma to the scrotum is an uncommon cause of testicular damage, but may need exploration, debride- ment and surgical repair. Sexual abuse needs to be considered in all genital injuries.
Recurrent scrotal pain in boys can be difficult to manage. Any associated symptoms or signs such as swelling or redness should be regarded as intermittent testicular torsion and the testes fixed.

40
Q

When does a normal foreskin retract?
At 1 year of age, about half of uncircumcised boys have a non-retractile (normal) foreskin. Only 1% of boys over 16 years old have a non-retractile foreskin. True or false

What acts a protection to the non-keratinised glanular and meatal squamous epithelium in an environment where astringent urine can cause inflammation or even ulceration. ? How can this be manifested? This needs to be differentiated from what? What signs are seen when there’s an infection?
What is the usually cause of the infection? What is a common cause of parental concern? What does it result from? Where else can this occur? When does it stop? How does sub preuptial smegma appear as? What does it look like? What is Paraphimosis? How is it treated? It has been regarded as an indication for what?

What happens in a normal retractile foreskin? What is seen a pathologically non retractile foreskin? What is the commonest condition that gives rise to a true phimosis ? What does this condition give rise to? How does this patient w this condition present?
What is the index indication for circumcision?

A

Abnormalities of the penis
The foreskin
A normal foreskin does not retract in infancy, and retraction should not be attempted.
The prepuce develops adherent to the underlying glans, and acts as protection to the non-keratinised glanular and meatal squamous epithelium in an environment where astringent urine can cause inflammation or even ulceration. This can be manifest as ammoniacal dermatitis (napkin rash) in infants and young children, where the preputial opening can be reddened and sore. It usually only needs reassur- ance and attention to routine hygiene. This needs to be differentiated from infection, or balanoposthitis, where the redness is more extensive, and, crucially, there is a purulent discharge. The infection is usually bacterial and needs antibiotic treatment, either topical or systemic. As it is rarely fungal, antifungal agents are not indicated. Topical corticosteroids may sometimes be beneficial. Ballooning of the foreskin on urination is a common cause of parental concern. It can look dramatic but seldom causes any trouble. It results from lysis of preputial adhesions around the glans before those at the preputial opening. Ballooning may also occur on the shaft of the penis, arising from the attachment of shaft skin below the coronal sulcus of the glans. Bal- looning stops when preputial adhesions have lysed completely. It has no functional consequence, does not represent obstruction, and does not need intervention.
Another cause of parental concern is sub-preputial smegma. It appears as a lump which grows briefly, seemingly under the non-retractile or partially retrac- tile foreskin. It is yellowish and malleable, and simply comprises desquamated skin and secretions. There is no need to intervene – it will discharge in due course (with typical appearance of smegma – ‘cottage cheese’; when the preputial adhesions break down.

Paraphimosis
This is a condition, usually in post-pubertal boys, of a retracted foreskin that cannot be reduced easily. There is a ring of narrower skin. The glans swells, and if the prepuce is not reduced it may result in compromise of the blood supply to the glans. Treat- ment (by reduction) is an emergency, which may require general anaesthesia. Paraphimosis has been regarded as an indication for circumcision, but this is no longer considered to be the case unless the foreskin is abnormal (as with BXO).

Non-retractile foreskin and phimosis
When traction is applied (gently) to a normal foreskin, the skin at the preputial opening is seen to evert, even if it does not necessarily open up .A foreskin that is pathologically non-retractile will not do this, and will truly render the glans ‘muzzled’, (Greek word ‘phimos’). This differentiates a foreskin that is simply non-retractile (i.e. normal, physiological) from one which is problematic. The commonest condition that gives rise to a true phimosis is balanitis xerotica oblit- erans, or BXO, which gives rise to progressive scarring which can extend onto the glans, into the meatus and ultimately into the urethra. Typically this affects older boys and young adults, and there is often a history that the foreskin was normally retractile in earlier child- hood. Figure 20.9 shows the typical appearance of BXO.
BXO is the index indication for circumcision, although there is some evidence that potent topical steroids, closely monitored, can cause it to regress.

41
Q

Name some medical reasons for circumcision
Name some complications of curcumcision
A non retractile foreskin is normal in which kids? Hypospadias arise from what ? What are the three features in this condition? There is rarely an associated or underlying disorder of sex development, and only very rarely another con- genital urinary tract abnormality. Investigation of the urinary tract with imaging is not routinely indicated. True or false
How is it managed? What is the ultimate functional sim of hypospadias surgery ? What are the commonest surgical complications?

Infants with hypospadias must not be circumcised, to preserve tissue for reconstruction. True or false

A

Circumcision
Circumcision remains a tradition in Jewish and Muslim religions.
Medical reasons for circumcision include:
• BXO causing a true phimosis
• recurrent balanoposthitis causing refractory
symptoms
• prophylaxis of recurrent urinary infection,
especially in the presence of a congenital uropathy (such as posterior urethral valves or vesicoureteric reflux) or if renal reserve is limited
• ifaccesstotheurethraisrequiredreliablyfor intermittent catheterization, e.g. spina bifida.
There are inevitably other indications for circum- cision, some of which are highly dependent on the individual family and surgeon. There is some evidence that circumcision affords protection against transmis- sion of HIV and HPV (human papillomavirus), and there are programmes promoting circumcision in newborn infants and young adult males in some countries with high prevalence of HIV infection.
There have been many techniques described for circumcision, and complications are uncommon. Up to
one boy in fifty has post-operative bleeding requiring a return to the operating theatre. Infection in the skin margin or ulceration of exposed granular skin may occur. Meatal stenosis can also occur, more often after circumcisions done for BXO, and this may require sub- sequent surgery. Rarer complications include urethral fistula.
A non-retractile foreskin is normal in preschool children

Hypospadias
. It is thought to arise from failure of development of ventral tissues of the penis – in particular failure of ventral urethral closure. For that reason it is really a constellation of ‘ventral hypoplasia’ of the penis.
Typically there are three features, although their occurrence is variable:
• a ventral urethral meatus–theurethralmeatusis variable in position but in most (80%) is on the distal shaft or glans penis (Fig. 20.11a)
• ventralcurvatureoftheshaftofthepenis(formerly called ‘chordee’) (Fig. 20.11b), more apparent on erection
• hoodedappearanceoftheforeskin–characteristic in appearance because of ventral foreskin deficiency but of no functional significance.

Management
Surgery is not mandatory, especially in a distal hypospadias when the penis and urinary stream are straight. However, it may be performed on functional or cosmetic grounds. The ultimate functional aim of hypospadias surgery is to allow a boy to pass urine in a straight line whilst standing, and to have a straight erection. Surgery, if needed, is usually performed in the first two to three years of life. The commonest surgical complications are breakdown of the repair or meatal narrowing. The prepuce may be preserved and recon- structed, although for more proximal hypospadias it is sometimes required for the repair itself. For this reason it is important that a boy with hypospadias is not circumcised before the repair.

42
Q

Variations in penoscrotal skin attachment and in the infant or child’s body habitus may make the penis look what? In summary: inguinak hernia causes what? When is Expedient surgical repair of inguinal hernias required? In an Undescended testis if the testis is palpable what is required? If it’s not palpable what is required?
If bilateral impalpable what is required? Why must torsion of the testis be considered?
Non retractile foreskin is pathological is associated with what?

A

Other conditions of the penis
Variations in penoscrotal skin attachment and in the infant or child’s body habitus may make the penis look buried. This is common with obesity, when the only treatment is weight loss, but improves with growth of the penis after puberty. However, it may persist if there is marked obesity.

Summary
Genital conditions in male infants
Inguinal hernia/hydrocele
• Cause inguinoscrotal swellings: clinically one can get above a hydrocele
• Expedient surgical repair of inguinal hernias is required in infants to prevent bowel strangulation or after reduction of an irreducible hernia
Undescended testis
• Common – up to 5% of term boys
• If testis palpable, requires orchidopexy
• If impalpable, may require laparoscopy to
establish presence of a testis
• If bilateral impalpable, urgent karyotype is
essential
• Retractile testes usually do not require surgery
Acute scrotal conditions
• May occur at any age but torsion of testis must be considered not only for acute pain of the scrotum but also for acute abdominal and groin pain
• scrotal surgical exploration is required unless torsion of the testis can be reliably excluded
Non-retractile foreskin
• Is normal in preschool children
• Is pathological if associated with BXO
• Circumcision is not recommended routinely but
is still traditional in some communities worldwide
Hypospadias
• Common – 1 in 200 boys
• Variable ventral urethral meatus and penile
curvature
• Surgery may be required in first two years of life
• Infants with hypospadias must not be circumcised

43
Q

What is the commonest problem in Vulvovaginitis?
Why does it occur in infants? If it’s infective what organism usually causes it? When will a vaginak discharge in vulvovaginitis indicate infection? What is the red flag symptom? What is an important cause of this symptom?
What is a characteristic appearance in fusion of the labia minora? The appearance sometimes raises parental concern about abnormal vaginal devel- opment, although these conditions are rare. Unless the labial adhesion causes significant symptoms, no specific treatment is required. True or false
What can be used to lyse the adhesions? Examination under anaesthesia, or formal ‘division of adhesions’ should be undertaken only exceptionally because of the high rate of recurrence. True or false

True obstruction or atresia of the vagina is rare. It may present with what In adolescence? And might coexist with what two signs? What usually reveals the cause?
If there is a bulging introitus that appears blue, the diagnosis is what? And what’s the treatment? Absence of this diagnosis represents a problem with what? In contrast to problems of the testes in boys, ovarian problems tend to be more difficult to diagnose because they are intra-abdominal. True or false

An ovarian cause for symptoms should be considered in a girl who presents with what signs?
In summary Vulvovaginitis in infants is usually due to? Labial adhesions tend to recur; no treatment is indicated unless symptomatic true or false

A

Genital disorders in girls

Vulvovaginitis/vaginal discharge
The commonest problem is redness of the vulva. In infants, this is often due to a nappy rash due to ammoniacal dermatitis. Less often, the vulvovaginitis is infective, occasionally with Candida infection. Vaginal discharge is common, and is usually innocuous unless it is green or offensive when it may indicate infec- tion. Foreign bodies are more often suspected than found; they are actually rare. The ‘red flag’ symptom is a bloody vaginal discharge, and needs referral to a specialist as vaginal rhabdomyosarcoma is a rare but important cause in preschool girls.

Labial adhesions
Fusion of the labia minora can be a cause of local irrita- tion in the prepubertal girl. There is usually an adequate orifice for the passage of urine. The characteristic appearance is of superficial fusion of the labia minora with a translucent (or even slightly bluish) area of flimsy tissue between the labia. Topical corticosteroids or oestrogens can be helpful to lyse the adhesions, especially if it allows the underlying introital anatomy to be seen, but readhesion is common.
Other conditions
True obstruction or atresia of the vagina is rare. It may present with primary amenorrhoea in adolescence.

This might co-exist with cyclical abdominal or pelvic pain representing obstruction to the flow of menses. Clinical examination usually reveals the cause. If there is a bulging introitus that appears blue, the diagnosis is imperforate hymen – and the treatment is hyme- notomy under anaesthesia. Absence of an imperforate hymen represents a problem of vaginal septation, canalization or more complex abnormality with para- mesonephric (Mullerian) duct development, and needs further imaging (often with MRI) to plan appropriate management.
An ovarian cause for symptoms should be considered in a girl who presents with acute abdominal pain (from ovarian cyst or torsion) or a mass (cyst or tumour).

Blood-stained vaginal discharge must be investigated
Summary
Genital conditions in female infants
• Vulvovaginitis in infants is usually due to nappy rash